Đến nội dung

dogsteven nội dung

Có 1000 mục bởi dogsteven (Tìm giới hạn từ 29-04-2020)



Sắp theo                Sắp xếp  

#690193 VMF's Marathon Hình học Olympic

Đã gửi bởi dogsteven on 11-08-2017 - 02:16 trong Hình học

Hướng chứng minh cho Bài toán 196:

Chứng minh $J$ là tâm nội tiếp và $LP$ vuông góc với $QR$ => $LG$ đi qua trực tâm $AEF$ và $AQR$ nên $LG$ vuông góc với đường thẳng Gauss.

@halloffame: Lời giải này chưa hoàn thiện, đầy đủ.




#690177 $n^p-p$ không chia hết cho q

Đã gửi bởi dogsteven on 10-08-2017 - 22:44 trong Số học

Bài toán mạnh hơn. Chứng minh tồn tại vô hạn số nguyên tố $q$ thỏa mãn.




#690119 Cho a + b = c, a >0, b > 0. Chứng minh rằng: $a ^{\fr...

Đã gửi bởi dogsteven on 10-08-2017 - 15:29 trong Bất đẳng thức - Cực trị

$m=\dfrac{2008}{2009}<1$. Khi đó áp dụng bất đẳng thức Bernoulli $\dfrac{c^m}{a^m}\leqslant \dfrac{a+mb}{a}<\dfrac{a+b}{a}$ 

Do đó $a^m>\dfrac{ac^m}{a+b}$, tương tự ta có $b^m>\dfrac{bc^m}{a+b}$ nên $a^m+b^m>c^m$




#690118 Trên bảng cho đa thức $x^8+x^7$. Mỗi lần đc phép nhân đa thức vs...

Đã gửi bởi dogsteven on 10-08-2017 - 15:15 trong Tổ hợp và rời rạc

Giả sử đến một lúc nào đó ta có đa thức $P(x)=a_nx^n+a_{n-1}x^{n-1}+...+a_1x+a_0$, ta sẽ chứng minh $(n-1)!.(na_{n}-a_{n-1})\vdots 8!-7!$

Với đa thức đầu tiên thì hiển hiên đúng.

Giả sử ta đang được đa thức $P(x)$ thỏa mãn tính chất trên.

Khi đó $P'(x)=na_n.x^{n-1}+(n-1)a_{n-1}.x^{n-2}+...+2a_2x+a_1$ cũng thỏa mãn vì $(n-2)!((n-1)na_n-(n-1)a_{n-1})=(n-1)!(na_n-a_{n-1})\vdots 8!-7!$

Và $P(x)(x+1)=a_nx^{n+1}+(a_n+a_{n-1})x^{n}+...+(a_1+a_0)x+a_0$ thỏa mãn vì $n!((n+1)a_n-a_n-a_{n-1})=n!(na_n-a_{n-1})\vdots 8!-7!$

 

Một lúc nào đó đến đa thức $ax+b$ thì $0!(1.a-b)\vdots 8!-7!$ nên $a-b\vdots 8!-7!\vdots 49$

 

Phương pháp làm: Spam nhiều vào sẽ ra!




#689883 Lấy E, D thuộc AC, AB sao cho PE$\perp$AB, PD$\perp...

Đã gửi bởi dogsteven on 08-08-2017 - 01:29 trong Hình học

Khúc đầu tiên có lẽ bạn trên làm không kỹ, những chỗ còn lại rất ổn. Ta có $\Delta BDP \sim \Delta PEC, \Delta HBP \sim \Delta HPC\Rightarrow DH\perp HE$

 

Từ $\Delta BDP\sim \Delta PEC \Rightarrow \dfrac{BD}{DN}=\dfrac{BP}{PC}$ suy ra $P, N$ liên hợp đẳng giác trong tam giác $ABC$

Do đó ta có thể nhìn bài toán như sau: Cho tam giác $ABC$ với hai điểm $P, Q$ liên hợp đẳng giác. $D$ là hình chiếu của $Q$ trên $BC$. $E, F$ là hình chiếu của $P$ trên $AC, AB$. $EF$ cắt $BC$ tại $S$. Chứng minh rằng $SP\perp AD$

Áp dụng các làm giống bạn trên đê giải quyết bài mở rộng cũng rất ổn.

Hoặc làm thế này chẳng hạn.  (Đối với bài toán chính và tương tự với bài toán mở rộng)

$SD.SE=SH.SK\Rightarrow S$ thuộc trục đẳng phương của đường tròn đường kính $AN$ và $NH$

Khi đó $SN\perp AH$




#689848 Bất đẳng thức lượng giác

Đã gửi bởi dogsteven on 07-08-2017 - 20:26 trong Bất đẳng thức - Cực trị

Anh Huyện có một cách phản chứng khá hay: Giả sử bất đẳng thức trên là sai. Khi đó nó lại mâu thuẫn với yêu cầu bài toán là chứng minh bất đẳng thức trên đúng. Do đó bất đẳng thức trên đúng (dpcm).

 

Em spam ạ!




#689814 Đề thi trại hè Hùng Vương 2017 - Khối 11

Đã gửi bởi dogsteven on 07-08-2017 - 15:20 trong Thi HSG cấp Tỉnh, Thành phố. Olympic 30-4. Đề thi và kiểm tra đội tuyển các cấp.

em cũng nghĩ Quay mà :)) không nghĩ đến Vector :)) mong anh giúp đỡ ạ

Xét phép $Q(x)=y\Leftrightarrow |x|=|y|$ và $(x,y)=\dfrac{\pi}{3}$

Khi đó ta có $2Q\left(\vec{MN'}\right)=Q\left(\vec{BA}\right)+Q\left(\vec{CC'}\right)=\vec{BB'}+\vec{CA}=2\vec{MP'}$

Do đó $MN'P'$ là tam giác đều, tương tự tam giác còn lại.




#689788 Tuần 2 tháng 8/2017: đường tròn $(D,DP)$ tiếp xúc với đường tròn...

Đã gửi bởi dogsteven on 06-08-2017 - 23:02 trong Chuyên mục Mỗi tuần một bài toán Hình học

Bài 1. Lấy $A'$ đối xứng với $A$ qua trung trực $BC$ ta suy ra $A', R, P$ thẳng hàng.

Mà $KO\perp AA'$, do đó $RP$ cắt $(O)$ tại $T$ khác $A'$ thì $D, T, L$ thẳng hàng và $DT=DP$

Tương tự như bước đầu của bạn ecchi123, ta có được $(AEF)$ cắt $(O)$ tại $S$ thì $\angle ASP=90^o$

$O'$ là tâm của $(AEF)$ thì $A', O', A$ thẳng hàng. Khi đó ta có góc $(SP, RK) = (SA, BC)=(SA, AO')=\angle O'AS = \angle STA'=(ST, RP)$
Do đó $\angle TSP = \angle PRK = \dfrac{\angle TDP}{2}$ nên $S$ thuộc $(D, DP)$
Ta có góc $(SD, BC)=2\angle STP = 2\angle O'AS = (SO', BC)$ nên $S, D, O'$ thẳng hàng. Suy ra $(D, DP)$ tiếp xúc với $(AEF)$



#689632 Z luôn thuộc $(J)$, là đường tròn bàng tiếp góc A của tam giác APQ

Đã gửi bởi dogsteven on 05-08-2017 - 16:09 trong Hình học

Bài này dùng hợp phép vị tự thôi bạn.




#689629 $DT$ đi qua điểm Lemoine của tam giác $ABC$.

Đã gửi bởi dogsteven on 05-08-2017 - 15:58 trong Hình học

Bổ đề 1 chứng minh làm sao ạ? Cho em xin cách giải bổ đề 1 ạ!!

$AS$ cắt $EF$ tại trung điểm $J$ thì $JTIS$ nội tiếp nên $ST\perp BC$




#689598 Đề thi trại hè Hùng Vương 2017 - Khối 11

Đã gửi bởi dogsteven on 05-08-2017 - 14:21 trong Thi HSG cấp Tỉnh, Thành phố. Olympic 30-4. Đề thi và kiểm tra đội tuyển các cấp.

Bài 2 liệu có thể giải quyết bằng phép Quay ?

Câu a có thể giải cực kỳ ngắn gọn bằng phép quay vector.




#689234 Đề thi trại hè Hùng Vương 2017 - Khối 11

Đã gửi bởi dogsteven on 01-08-2017 - 17:55 trong Thi HSG cấp Tỉnh, Thành phố. Olympic 30-4. Đề thi và kiểm tra đội tuyển các cấp.

Tổ hợp. Với hai tập $A, B$, ta định nghĩa $A+B=\{a+b| a\in A, b\in B\}$. Khi đó dễ thấy $|A+B|\leqslant |A|.|B|$

Thấy rằng $\forall x\in \{0, 1, 2, ..., 10^{2017}-1\}$ thì $x\in A_1+A_2+...+A_{2017}$ nên $|A_1+A_2+...+A_{2014}|\geqslant 10^{2017}$

Ta có $|A_1+A_2+...+A_{2017}|\leqslant |A_1|.|A_2+A_3+...+A_{2017}|\leqslant ... \leqslant |A_1|.|A_2|.....|A_{2017}|=k^{2017}$

Do đó $k\geqslant 10$. Với $k=10$,  đặt $A_i=\{0, 10^{i-1}, 2.10^{i-1}, ..., 9.10^{i-1}\}$

Do $x\in \{0, 1, 2, ..., 10^{2017}-1\}$ thì $x<10^{2017}$ nên có thể viết $x=a_1+a_2.10+a_3.10^2+...+a_{2017}.10^{2016}$ với $a_i\in \{0, 1, ..., 9\}$

Và hiển nhiên $a_i.10^{i-1}\in A_i$ nên ta có điều phải chứng minh.

Vậy $k$ bé nhất là $10$




#689131 Tuần 1 tháng 8/2017: $AU,BV,CW$ đồng quy

Đã gửi bởi dogsteven on 30-07-2017 - 23:10 trong Chuyên mục Mỗi tuần một bài toán Hình học

Bài 1. Đặt $\dfrac{LX}{XA}=k, d=AL, e=BL, z=CL$. Khi đó $\dfrac{1}{k}\vec{LD}+\vec{BD}+\vec{CD}=\vec{0}$

Do đó mà $ka^2\vec{LU}+f^2\vec{BU}+e^2\vec{CU}=\vec{0}$

Như vậy:

$\vec{BU}=\dfrac{(ka^2+e^2)\vec{BL}-e^2\vec{CL}}{ka^2+e^2+f^2}$

$\vec{CU}=\dfrac{(ka^2+f^2)\vec{CL}-f^2\vec{BL}}{ka^2+e^2+f^2}$

$\vec{AU}=\vec{AL}+\vec{LU}=\dfrac{-b^2\vec{BL}-c^2\vec{CL}}{a^2}+\dfrac{-f^2\vec{BL}-e^2\vec{CL}}{ka^2+e^2+f^2}$

Ta sẽ tìm $y,z$ thỏa mãn: $\vec{AU}+y\vec{BU}+z\vec{CU}=\vec{0}$ hay

$\dfrac{-b^2}{a^2}+\dfrac{y(ka^2+e^2)-(z+1)f^2}{ka^2+e^2+f^2}=-\dfrac{c^2}{a^2}+\dfrac{z(ka^2+f^2)-(y+1)e^2}{ka^2+e^2+f^2}=0$

Do đó $y=\dfrac{ka^2b^2+f^2(a^2+b^2+c^2)}{ka^4}, z=\dfrac{ka^2c^2+e^2(a^2+b^2+c^2)}{ka^4}$

Mà $a^2\vec{AL}+b^2\vec{BL}+c^2\vec{CL}=\vec{0}$ nên

$2c^2a^2-(a^2d^2+b^2e^2+c^2f^2)=e^2(a^2+b^2+c^2)$ và $2a^2b^2-(a^2d^2+b^2e^2+c^2f^2)=f^2(a^2+b^2+c^2)$

Do đó $\dfrac{y}{z}=\dfrac{(k+2)a^2b^2-(a^2d^2+b^2e^2+c^2f^2)}{(k+2)c^2a^2-(a^2d^2+b^2e^2+c^2f^2)}$

Tương tự với các cạnh còn lại và áp dụng định lý Ceva cho ta điều phải chứng minh.




#689001 $\sqrt[n]{2+\sqrt{3}}+\sqrt[n]{2...

Đã gửi bởi dogsteven on 29-07-2017 - 15:53 trong Đa thức

$a=\sqrt[n]{2+\sqrt{3}}$ và $b=\sqrt[n]{2-\sqrt{3}}$. Khi đó $a^n+b^n=4$ và $ab=1$

Lúc này ta có thể viết như sau:

$(a+b)^n=P_1(a^{n-2}+b^{n-2}, a^{n-4}+b^{n-4},...)$

$(a+b)^{n-2} =P_2(a^{n-4}+b^{n-4}, a^{n-8}+b^{n-8},...)$

...

Trong đó các $P_i$ là hàm tuyến tính bậc nhất hệ số nguyên. Từ phương trình dưới cùng ta có thể biểu diễn $a^{n-2k}+b^{n-2k}$ (với $n-2k$ là nhỏ nhất) theo một đa thức hệ số hữu tỉ $a+b$

Từ đó tiếp tục thế lên các phương trình phía trên ta được điều phải chứng minh. Ngoài ra phương án này còn chỉ ra đa thức đó còn có bậc là $n$




#688994 $P(t)=Q(t^{2}-1,t^{3}-1)$

Đã gửi bởi dogsteven on 29-07-2017 - 14:05 trong Đa thức

$Q$ là hàm số hay là đa thức vậy bạn. Nếu là đa thức thì xét $P(x)=x^3-x^2-x+2$ thì sao tìm được $Q$ đây bạn.




#688968 Chứng minh rằng $m|ak!$.

Đã gửi bởi dogsteven on 29-07-2017 - 03:07 trong Đa thức

(a) Ta có $P(x)=a_0(x-1)...(x-k)+a_1(x-1)...(x-k+1)+...+a_k$ theo nội suy Newton thì dễ dàng quy nạp rằng $m|a_i.(k-i)!$
(b) Chọn $P(x)=a(x-1)...(x-k)+mQ(x)$ trong đó $Q$ hệ số nguyên và bậc nhỏ hơn $k$



#688578 Đề luyện tập olympic khối 11 VMF tuần 4 tháng 7

Đã gửi bởi dogsteven on 25-07-2017 - 02:05 trong Thi HSG cấp Tỉnh, Thành phố. Olympic 30-4. Đề thi và kiểm tra đội tuyển các cấp.

Bài 1. Giả sử $\lim x_{n+1}+ax_{n} = 0$

Khi đó $\forall \epsilon >0$, tồn tại $N$ để $\forall n>N$, ta được: $|x_{n+1}+ax_{n}|<\epsilon$

Do đó $|x_{n+1}|<|a.x_{n}|+\epsilon<...<|a|^{t}.|x_{n_0}|+(1+|a|+|a^2|+...+|a^{t-1}|)\epsilon<|a|^{t}.|x_{n_0}|+\dfrac{\epsilon}{1-|a|}$

Cho $n$ tiến tới vô cực ta suy ra $|x_n|<\epsilon$, suy ra $\lim x_n=0$




#688014 58th IMO 2017

Đã gửi bởi dogsteven on 19-07-2017 - 13:27 trong Thi HSG Quốc gia và Quốc tế

 

i. Con thỏ di chuyển bí mật tới điểm $A_n$ mà $A_{n-1}$ cách $A_n$ đúng $1$.

iii. Thợ săn di chuyển tới điểm $B_n$ sao cho $B_n$ cách $B_{n-1}$ đúng $1$.

 

Em ggwp cách dịch ạ.




#687827 Truần 3 tháng 7/2017: đường tròn ngoại tiếp tam giác $GDP$ đi qua...

Đã gửi bởi dogsteven on 17-07-2017 - 16:07 trong Chuyên mục Mỗi tuần một bài toán Hình học

Lời giải bài 2. 

Gọi $R, J$ là điểm chính giữa cung lớn, cung nhỏ $BC$ của $(ABC)$. Gọi $T$ là tâm của $(RKD)$

Ta có: $IJ.IR = IB.IC = IP.ID$ nên $P, J, D, R$ đồng viên.

Do đó $\angle ART = \angle KRT = 90^o - \angle PDT = 90^o - \angle AJR =\angle ARJ$ nên $T$ thuộc trung trực $BC$

Như vậy $TM=TD$ nên $K, M, D, R$ đồng viên. Mà $R$ là điểm chính giữa cung $MD$ nên $KA$ là phân giác $MKD$.




#687185 Tuần 2 tháng 7/2017: Chứng minh rằng $PQ$ và $IF$ cắt nha...

Đã gửi bởi dogsteven on 10-07-2017 - 22:53 trong Chuyên mục Mỗi tuần một bài toán Hình học

Bài 2.

Bổ đề. Cho tam giác $ABC$ và hai điểm $E, F$ thỏa mãn $AE, AF$ đẳng giác góc $BAC$. $BE$ cắt $CF$ tại $P$, $BF$ cắt $CE$ tại $Q$. Khi đó $AP, AQ$ đẳng giác góc $BAC$

Chứng minh.

Biến đổi tỉ số kép: $A(EP, BC)=C(EP, BA)=C(QF, BA)=A(QF, BC)=A(FQ, CB)$ nên $AP, AQ$ đẳng giác góc $BAC$

 

Hệ thức Maclaurin mở rộng: Cho $A, B, C, D, E$ nằm trên một đường thẳng, khi đó: $\overline{AB}.\overline{AC}=\overline{AD}.\overline{AE}\Leftrightarrow \dfrac{\overline{AB}}{\overline{AC}}=\dfrac{\overline{DB}}{\overline{DC}}.\dfrac{\overline{EB}}{\overline{EC}}$

 

Áp dụng bổ đề trên cho tam giác $APP*$ với $AQ, AQ*$ đẳng giác góc $PAP*$ ta suy ra $AR, AR*$ đẳng giác góc $PAP*$ nên đẳng giác góc $BAC$

Tương tự với các góc còn lại ta suy ra $R, R*$ đẳng giác trong tam giác $ABC$

Gọi $P_a, P*_a, Q_a, Q*_a, R_a, R*_a$ là hình chiếu của $P, P*, Q, Q*, R, R*$ trên $BC$

Ta sẽ chứng minh: $\dfrac{\overline{R_aP_a}}{\overline{R_aP*_a}}.\dfrac{\overline{R*_aP_a}}{\overline{R*_aP*_a}}=\dfrac{\overline{Q_aP_a}}{\overline{Q_aP*_a}}.\dfrac{\overline{Q*_aP_a}}{\overline{Q*_aP*_a}}$

Nhóm theo từng cặp trên các hình thang, điều này tương đương với: $\dfrac{\overline{RP}}{\overline{QP}}.\dfrac{\overline{R*P}}{\overline{Q*P}}=\dfrac{\overline{RP*}}{\overline{Q*P*}}.\dfrac{\overline{R*P*}}{\overline{QP*}}$, điều này hiển nhiên đúng.

Gọi $W$ là giao điểm của trục đẳng phương của hai đường tròn pedal ứng với $P, Q$ và $BC$

Khi đó $\overline{WP_a}.\overline{WP*_a}=\overline{WQ_a}.\overline{WQ*_a}$ nên theo hệ thức Maclaurin mở rộng, ta có:

$$\dfrac{\overline{WP_a}}{\overline{WP*_a}}=\dfrac{\overline{Q_aP_a}}{\overline{Q_aP*_a}}.\dfrac{\overline{Q*_aP_a}}{\overline{Q*_aP*_a}}=\dfrac{\overline{R_aP_a}}{\overline{R_aP*_a}}.\dfrac{\overline{R*_aP_a}}{\overline{R*_aP*_a}}$$

Vậy theo hệ thức Maclaurin mở rộng ta lại có: $\overline{WP_a}.\overline{WP*_a}=\overline{WR_a}.\overline{WR*_a}$

Do đó $W$ là tâm đẳng phương của đường tròn pedal của ba điểm $P, Q, R$

Trục đẳng phương của hai đường tròn pedal ứng với $P, Q$ phải cắt ít nhất một cạnh nữa của tam giác $ABC$

Làm tương tự với cạnh đó ta được điều phải chứng minh.




#687125 Tuần 2 tháng 7/2017: Chứng minh rằng $PQ$ và $IF$ cắt nha...

Đã gửi bởi dogsteven on 10-07-2017 - 14:39 trong Chuyên mục Mỗi tuần một bài toán Hình học

Bài 1. Gọi $G$ là giao điểm của $FI$ với $(O)$. $AD$ cắt $BC$ tại $D'$.

$DG$ cắt $AE$ tại $T$ thì $IT || BC$. Do đó $\dfrac{TE}{TA}=\dfrac{ID'}{IA}=\dfrac{BC}{AB+AC}=\dfrac{BD}{DA}=\dfrac{DI}{DA}$

Do đó $DT$ chia đôi $IE$ tại trung điểm $R$ nên $D(GQ, PA)=-1$ mà $GFDN$ là tứ giác điều hòa nên $A(GQ, PD)=-1$ hay $D(GQ, PA)=A(GQ, PD)$

Do đó $P, Q, G$ thẳng hàng nên ta có điều phải chứng minh.




#687006 USA TSTST 2017

Đã gửi bởi dogsteven on 09-07-2017 - 00:27 trong Thi HSG Quốc gia và Quốc tế

Bài 5.

Đầu tiên ta thấy $P$ là tâm vị tự trong của $\omega_B, \omega_C$.$C$ là tâm vị tự ngoài của $\omega_C, (I).$

Do đó theo định lý Monge D'Alambert thì tâm vị tự trong của $\omega_B, (I)$ nằm trên $CP.$

Mà tâm vị tự trong của $\omega_B, (I)$ phải nằm trên $BI$ nên nó chính là $X.$

Tương tự với $Y$ là tâm vị tự trong của $\omega_C, (I).$

$T$ là tiếp điểm của $(I)$ với $BC$ và $S$ đối xứng với $T$ qua $I.$

Do tính chất vị tự của hai đường tròn nên $S, X, E$ thẳng hàng, tương tự $S, Y, F$ thẳng hàng.




#686717 [Hình học] THPT tháng 11: $UK$ đi qua điểm cố định khi $P, Q...

Đã gửi bởi dogsteven on 06-07-2017 - 17:54 trong Thảo luận đề thi VMEO IV

Dù bài này lâu rồi, nhưng em mới tìm được lời giải qua ý tưởng dùng trục đẳng phương của một bạn nào đó trên facebook.

Gọi $R$ là trực tâm tam giác $ADE$. $B', C'$ là hình chiếu của $B, C$ trên $DE$ và $H$ là trực tâm tam giác $ABC$

Ta có $\Delta GUE \sim \Delta BDP$ và $\Delta GUD\sim \Delta CEP$, suy ra $\dfrac{UE}{UD}=\dfrac{PD}{DB}.\dfrac{EC}{EP}$

Ta còn có $\Delta EC'C\sim \Delta PDA$ và $\Delta DB'B\sim \Delta PEA$ nên $\dfrac{EC'}{DB'}=\dfrac{EC'}{EC}.\dfrac{DB}{DB'}.\dfrac{EC}{DB}=\dfrac{PD}{DB}.\dfrac{EC}{EP}=\dfrac{UE}{UD}$

Do đó $U$ thuộc trục đẳng phương của hai đường tròn đường kính $BE$ và $CF$

Mà $RH$ là trục đẳng phương của đường tròn đường kinh $BE$ và $CF$ nên $HU$ đi qua $R$.

$R$ đối xứng với $L$ qua $EF$ nên $R, U, K$ thẳng hàng. Do đó $UK$ đi qua $H$




#686479 Đề OLYMPIC GẶP GỠ TOÁN HỌC 2017 KHỐI 12

Đã gửi bởi dogsteven on 04-07-2017 - 17:10 trong Thi HSG cấp Tỉnh, Thành phố. Olympic 30-4. Đề thi và kiểm tra đội tuyển các cấp.

Bài 3.

(a) Gọi $A_1, A_2$ là điểm chính giữa cung $BC$ của $(O)$ và $Z$ là trung điểm $OB$

Khi đó $OT.OS=2OZ.OB=OB^2\Rightarrow (A_1A_2,ST)=-1$

(b) Dễ thấy $AE, AF$ đối xứng nhau qua phân giác góc $A$, $EF || BC$

Ta có $BA_1$ là phân giác góc $TBS$ mà $BA_1$ là phân giác góc $TBF$ nên $SB$ đi qua $F$, tương tự $SC$ đi qua $E$

Áp dụng định lý Pascal cho $CCBFAE$ ta được $X\in AF$, tương tự $Y\in AE$




#686337 Tuần 1 tháng 7/2017: Trung điểm của $QR$ nằm trên đường tròn ngoại...

Đã gửi bởi dogsteven on 03-07-2017 - 13:21 trong Chuyên mục Mỗi tuần một bài toán Hình học

Bài 1.

Gọi $K, L$ là điểm chính giữa cung $BC$ chứa $A$ và không chứa $A$ của $(O)$, ta có $KX\perp QR$.

Do đó ta cần chứng minh $PD$ đi qua $L$. Ta còn có $L$ là giao điểm hai tiếp tuyến tại $B$ và $C$ của $(IBC)$

Gọi $B', C'$ là giao điểm của $BI, CI$ với $EF$, ta có $\widehat{BB'C}=\widehat{BC'C}=90^o$, ta chuyển về bài toán sau.

Cho tam giác $ABC$ với ba đường cao $AD, BE, CF$ đồng quy tại trực tâm $H$. $T$ là giao điểm hai tiếp tuyến tại $B, C$ của $(O)$. $M$ là trung điểm $BC$.

$HM$ cắt $EF$ tại $S$. Chứng minh rằng $D, T, S$ thẳng hàng.

 

Đường tròn đường kính $AH$ cắt $(O)$ tại $K$ khác $A$, khi đó $K, H, M, S$ thẳng hàng và $(KH, SM)=-1$

$AD$ cắt $(O)$ tại $D'$ khác $A$ thì $A(KD', BC)=-1$ nên $KD'$ đi qua $T$. Mà $D$ là trung điểm $HD'$ nên $KD$ chia đôi $MT$

Do đó $D(KA, MT)= -1 = D(KH, MS)\Rightarrow D, S, T$ thẳng hàng.